Use app×
Join Bloom Tuition
One on One Online Tuition
JEE MAIN 2025 Foundation Course
NEET 2025 Foundation Course
CLASS 12 FOUNDATION COURSE
CLASS 10 FOUNDATION COURSE
CLASS 9 FOUNDATION COURSE
CLASS 8 FOUNDATION COURSE
0 votes
168 views
in General by (115k points)
closed by
For an incompressible flow field, \(\vec V\) , which one of the following conditions must be satisfied?
1. \(\nabla \cdot \vec V = 0\)
2. \(\nabla \times \vec V = 0\)
3. \(\left( {\vec V \cdot \nabla } \right)\vec V = 0\)
4. \(\frac{{\partial \vec V}}{{\partial t}} + \left( {\vec V \cdot \nabla } \right)\vec V = 0\)

1 Answer

0 votes
by (152k points)
selected by
 
Best answer
Correct Answer - Option 1 : \(\nabla \cdot \vec V = 0\)

Explanation:
Any flow must satisfy the continuity equation if continuity equation violates then such flow is not possible.

continuity equation for incompressible flow is

\(\frac{{\partial u}}{{\partial x}} + \;\frac{{\partial v}}{{\partial y}} + \;\frac{{\partial w}}{{\partial z}} = 0\) which is \(\nabla \cdot \vec V = 0\)

∴ For an incompressible flow field, divergence must be zero, i.e. \(\nabla \cdot \vec V = 0\)

Welcome to Sarthaks eConnect: A unique platform where students can interact with teachers/experts/students to get solutions to their queries. Students (upto class 10+2) preparing for All Government Exams, CBSE Board Exam, ICSE Board Exam, State Board Exam, JEE (Mains+Advance) and NEET can ask questions from any subject and get quick answers by subject teachers/ experts/mentors/students.

Categories

...